What is Limit: Definition and 1000 Discussions

In mathematics, the limit inferior and limit superior of a sequence can be thought of as limiting (i.e., eventual and extreme) bounds on the sequence. They can be thought of in a similar fashion for a function (see limit of a function). For a set, they are the infimum and supremum of the set's limit points, respectively. In general, when there are multiple objects around which a sequence, function, or set accumulates, the inferior and superior limits extract the smallest and largest of them; the type of object and the measure of size is context-dependent, but the notion of extreme limits is invariant.
Limit inferior is also called infimum limit, limit infimum, liminf, inferior limit, lower limit, or inner limit; limit superior is also known as supremum limit, limit supremum, limsup, superior limit, upper limit, or outer limit.

The limit inferior of a sequence




x

n




{\displaystyle x_{n}}
is denoted by





lim inf

n





x

n




or





lim
_



n






x

n


.


{\displaystyle \liminf _{n\to \infty }x_{n}\quad {\text{or}}\quad \varliminf _{n\to \infty }x_{n}.}
The limit superior of a sequence




x

n




{\displaystyle x_{n}}
is denoted by





lim sup

n





x

n




or





lim
¯



n






x

n


.


{\displaystyle \limsup _{n\to \infty }x_{n}\quad {\text{or}}\quad \varlimsup _{n\to \infty }x_{n}.}

View More On Wikipedia.org
  1. J

    MHB What is the Limit as n Approaches Infinity of the Integration of Cosine squared?

    Finding $$\lim_{n\rightarrow \infty}\sqrt{n}\int^{\frac{\pi}{4}}_{0}\cos^{2n-2}(z)dz$$
  2. A

    A What is the limit of this (complicated) set?

    This is going to take a while to set up, so I apologize for that. This came up in the course of thinking about the Strong Law of Large Numbers. It's not homework. Suppose you have a doubly infinite sequence of random variables X_{i,n} that obey the following almost sure convergence relations...
  3. J

    I The Classical Limit of Maxwell-Boltzmann Distribution

    I have been reading about the quantum effects that limit the Maxwell-Boltzmann Distribution under certain conditions which leads to the Bose-Einstein or Fermi-Dirac Distribution. I have difficulty grasping the reasons why these quantum-effects occur only at certain conditions and why exactly...
  4. X

    I Take a limit in this 2 equation system of 1st order ODEs

    Hello, I'm having a problem with this system. Ignore the physics. I have the feeling it should be tremendously easy... but I can't figure it out. I don't know how to extract it from the pdf so I'll post just the these 2 pages. https://ufile.io/39ovq The equations are (1.14) and (1.15), the...
  5. L

    What is true for limit of f (x,y) as (x,y)→(0,1)?

    Homework Statement Let f be a function from R2 to R. Suppose that f (x, y) → 3 as (x, y) approaches (0,1) along every line of the form y = kx + 1. What can you say about the limit lim(x,y)→(0,1) f (x, y)? Check the box next to the correct statement. Homework Equations N/A The Attempt at a...
  6. benorin

    B Would there be an upper limit to density in QM?

    Admitted I know very little about QM, but I've been thinking about black holes and I wondered if there would be an upper limit to density of an object of the smallest size allowable if the particles are not being observed by anyone (since black holes are black)? I ignorantly wondered that...
  7. S

    You're welcome! Good luck with your project.

    First time poster here, thanks in advance! I have a project I'm working on, and I'm looking for a way to limit the rotational motion of a shaft inside a cylinder. The cylinder is fixed, and the shaft is spinning inside the cylinder coaxially. Basically, torque will be applied to the shaft...
  8. V

    MHB  Limit of $x_n$ Sequence: $\pi/2$

    Let $x_{0}=1$ and $x_{n+1}=(-1)^{n}(\frac{\pi }{2}-\arctan(\frac{1}{x_{n}}))$ I have the following options to choose from: 1. $x_n$ is unbounded 2. $x_n$ is increasing and the limit of $x_n$ is $1$ 3. the limit of $x_n$ is $\pi/2$. 4. the limit of $x_n$ is $0$ My attempt: I used...
  9. hnnhcmmngs

    Limit of a multivariable function

    Homework Statement If possible, calculate the following limit: \lim_{(x,y)\rightarrow (0,0)} {\frac{2x^2 + 3y^2}{5xy}} Homework Equations N/A The Attempt at a Solution [/B] I tried using both parametric and polar equations to find the limit, but neither worked. Setting either x or y...
  10. navneet9431

    What Is the Limit of (x - tan(x)) / x³ as x Approaches 0?

    <Moderator's note: Moved from a technical forum and thus no template.> $$\lim_{x\rightarrow 0} (x-tanx)/x^3$$ I solve it like this, $$\lim_{x\rightarrow 0}1/x^2 - tanx/x^3=\lim_{x\rightarrow 0}1/x^2 - tanx/x*1/x^2$$ Now using the property $$\lim_{x\rightarrow 0}tanx/x=1$$,we have ...
  11. V

    MHB Why does this limit not exist?

    Why the following limit doesn't exists ? $$\lim_{x\rightarrow 0}xe^{-\frac{1}{x}}$$ I think it's because of $\frac{1}{x}$ which doesn't exists, right ?
  12. D

    I How Does Schwartz Derive the Schrödinger Equation from QFT?

    In Matthew Schwartz's QFT text, he derives the Schrodinger Equation in the low-energy limit. I got lost on one of the steps. First he mentions that $$ \Psi (x) = <x| \Psi>,\tag{2.83}$$ which satisfies $$i\partial _t\Psi(x)=i\partial_t< 0|\phi...
  13. J

    MHB Evaluate Limit: $\frac{0}{0}$ - Can Someone Help?

    I'm trying to show... $\lim_{{x}\to{0^+}}\left(\frac{e^(\frac{-1}{x})}{x^n}\right)=0$ I guess my calculus is a bit rusty. Can someone help me out? Here's what I've got...
  14. V

    MHB Calculating the Limit of Sequence $(y_n)$ with $(x_n)$ Limit = $\frac{\pi^2}{6}$

    I have the following sequence $(x_{n})$ , $x_{n}=1+\frac{1}{2^{2}}+...+\frac{1}{n^{2}}$ which has the limit $\frac{\pi ^{2}}{6}$.I need to calculate the limit of the sequence $(y_{n})$, $y_{n}=1+\frac{1}{3^{2}}+...+\frac{1}{(2n-1)^{2}}$ I don't know how to start.I think I need to solve the limit...
  15. Arman777

    B Naked Singularity, Black hole mass limit

    I came across a question on PSE. I am not sure its a violation to ask the same question here, but there's no answer to the question in there so I wanted to ask it here. Quoting his question,"Since the universe has a positive cosmological constant, there is an upper limit on the mass of the...
  16. V

    MHB Limit of $(x_{n})_{n\geq 1} with Given Conditions

    Hi! I have the following sequence $$(x_{n})_{n\geq 1}, \ x_{n}=ac+(a+ab)c^{2}+...+(a+ab+...+ab^{n})c^{n+1}$$ Also I know that $a,b,c\in \mathbb{R}$ and $|c|<1,\ b\neq 1, \ |bc|<1$ I need to find the limit of $x_{n}$. My attempt is in the picture.The result should be $\frac{ac}{(1-bc)(1-c)}$ I...
  17. V

    MHB Evaluating Limit $$\frac{\ln2}{2}+\cdots+\frac{\ln n}{n}$$

    Hi, $$\lim_{n \to \infty}\dfrac{\dfrac{\ln2}{2}+\dfrac{\ln3}{3}+\cdots+\dfrac{\ln n}{n}}{\ln^2 n}.$$ After I applied Stoltz-Cesaro I got $$\lim_{n \to \infty}\dfrac{\dfrac{\ln2}{2}+\dfrac{\ln3}{3}+\cdots+\dfrac{\ln n}{n}}{\ln^2 n}=\lim_{n \to \infty}\dfrac{\dfrac{\ln (n+1)}{n+1}}{\ln^2...
  18. V

    MHB Square root n limit ( sum question )

    Hi! $$(x_{n})_{n\geq 2}\ \ x_{n}=\sqrt[n]{1+\sum_{k=2}^{n}(k-1)(k-1)!}$$ $$\lim_{n\rightarrow \infty }\frac{x_{n}}{n}=?$$ I know how to solve the limit but I don't know how to solve the sum $\sum_{k=2}^{n}(k-1)(k-1)!$ which should be $(n! - 1)$ The limit would become $\lim_{n\rightarrow \infty...
  19. A

    Solving Limits: A Shortcut to Finding Limits Using the Conjugate Theorem

    <Moderator's note: Moved from a technical forum and thus no template.> How to find this limit? \lim_{x \to 0} \frac{5x} {3 -\sqrt{9-x}} I'd tried to find this limit as below but the result is 0: \lim_{x \to 0} \frac{5x} {3 -\sqrt{9-x}} \lim_{x \to 0} \frac{5x} {3 - \sqrt{9-x}} × \frac{3 +...
  20. A

    How Do You Solve the Limit Using the Quotient Law?

    Homework Statement lim (1/x - 1/3) / (x-3) x->3 Homework EquationsThe Attempt at a Solution I tried to cancel the bottom (x-3) out by multiplying the top by 3/3 and x/x and then got ((3-x)/3x)/(x-3) but ended with 0/0 and the right answer is -1/9. The top part is confusing me.
  21. V

    MHB Find Limit of Sequence $(a_{n})$: $a_{2n+1}$

    I have the following sequence $(a_{n})$, $a_{1}=1$ $$a_{n+1}=\begin{cases} a_{n}+\frac{1}{2} & \text{ if } n \ is \ even \\ \frac{a_{n}}{3} & \text{ if } n \ is \ odd \end{cases}$$ I need to find $$\lim_{n\rightarrow \infty }a_{2n+1}$$ I tried something but I didn't get too far.I rewrite the...
  22. A

    How can finding limits by conjugates be used to solve for rational expressions?

    The other day in a fit of boredom I decided to dust off my old math books (high school and undergrad) and see if I can still do basic calculus. These days if I need to solve anything I ask a computer to do it, the hazards of getting a job in industry I suppose. All that said, I have been...
  23. V

    MHB Why Does $$\lim_{n\rightarrow \infty }\frac{n^{2016}\cdot 2^{n-1}}{3^{n}}=0$$?

    Why $$\lim_{n\rightarrow \infty }\frac{n^{2016}\cdot 2^{n-1}}{3^{n}}=0$$ ? Because $3^{n}> 2^{n-1} $ ?
  24. Y

    MHB Limit with x in both base and exponenet

    Dear all, I am trying to solve the following limit: \[\lim_{x\rightarrow 0}(e^{ax}+x)^{\frac{1}{x}}\] where \[a\] is a constant. I know that the limit is equal to \[e^{a+1}\] but not sure how to prove it. Thank you.
  25. V

    MHB Find the Limit of a Sequence: Tips & Techniques

    I have the sequence from the picture and I have to demonstrate that this sequence has a limit. I always get stuck at this kind of exercises.How to approach an exercise like this?
  26. navneet9431

    Evaluating the Limit of Cosine Function Using L'Hospital's Rule - Explained

    <Moderator's note: Moved from a technical forum and thus no template.> $$\lim_{x \to 0} \cos(\pi/2\cos(x))/x^2$$ I tried to evaluate the limit this way, $$\lim_{x \to 0} \cos(\pi/2\cdot1)/x^2$$ since $$\cos0=1$$ $$\lim_{x \to 0} \cos(\pi/2\cdot1)/x^2=\lim_{x \to 0} 0/x^2$$ Now apply...
  27. Specter

    Limit problem not making sense

    Homework Statement ##\frac {lim} {x→0} \frac {\sqrt {x+1}-1} {x}## Homework EquationsThe Attempt at a Solution I know the limit as x approaches 0 isn't supposed to be a fraction but I can't get the x approaches 0 under the lim. I couldn't get some of this typed out in latex, it just wouldn't...
  28. V

    MHB Solving Limits of Integrals: Advice & Tips

    I tried to use integration by parts. I took f(x)=arctan(x) => f'(x)= 1/x^2+1 g'(x)=cos(nx) => g(x)= sin(nx)/n So I get sin(nx)/n * arctan(x) - integral from 0 to 1 from sin(nx)/n(x^2+1) How to continue ? I'm always getting stuck with this kind of exercises ( limits of integrals ) because I don't...
  29. A

    MHB Problem about limit to infinity

    \lim_{{T}\to{\infty}}N \bar{h}\omega \left( \frac{1}{2} + \frac{1}{e^{\frac{ \bar{h}\omega}{k_BT}}-1} \right) In term \lim_{{T}\to{\infty}}N \bar{h}\omega \left( \frac{1}{e^{\frac{ \bar{h}\omega}{k_BT}}-1} \right)=N \bar{h}\omega \lim_{{T}\to{\infty}}\left( \frac{1}{e^{\frac{...
  30. V

    MHB Limit (x1) as m->infinity ( x1 - the lowest root )

    I have the following equation: x^2 - 2(m+1)x + 3m + 1=0 Also, I know that x1 is the lowest root of this equation. I need to solve lim (x1) as m->infinity A. 1 B. 3/2 C. 0 D. -1/2 E. -1 I tried to solve the equation with the discriminant then to calculate the limit but didn't work. Also, I think...
  31. Adgorn

    Convergence of Roots at Infinity

    Homework Statement Hi everyone, I'm currently making my way through Spivak's calculus and got stuck in question 41 of chapter 5. It's important to note that at this point, the book has only reached the subject of limits (haven't reached continuous functions, derivatives, integrals, series...
  32. S

    I Precise intuition about limits and infinitesimals

    I've understood the formal definition of limits and its various applications. However, I'm trying to dive more into the history of how the concept of limits were conceived (more than what Wikipedia tends to cover), and how to formally understand and visualise infinitesimals. For example, I know...
  33. A

    I Calculating the resolution limit using Fermat's principle

    Hi, I read the Feynman Lectures Volume 1, Chapter 27, section 27-7, which can be here. In the lecture he describes the fundamental limits of resolution and provides a criterion. Here is the diagram I am referring to, figure 27.-9: There are two light sources, ##P## and ##P'## There is an...
  34. Krushnaraj Pandya

    Solving a Limit Problem using L'Hospital Rule

    Homework Statement ## \lim x-a \frac {{a^x-{x^a}}}{{x^x}-{a^a}} = -1## then a is? Homework Equations L'Hospital rule The Attempt at a Solution Using LHR we can write numerator as ##\frac{a^x ln{a}-ax^{a-1}}{x^xln(x+1)}## plugging x=a and equating to -1 gives 1-ln(a)=ln(a+1); so 1=ln(a(a+1))...
  35. Krushnaraj Pandya

    Rules to apply L'Hospital on a limit

    Homework Statement ## \lim x-0 \frac {xcosx-log(1+x)}{x^2}## Homework Equations ##\frac{log(1+x)}{x}=1## ...(i) The Attempt at a Solution Using (i) we can write numerator as xcosx-x, cancelling x from denominator we have cosx-1/x, this is 0/0 form so we can use LHR which gives us -sinx/1 but...
  36. Krushnaraj Pandya

    Limit of 0^0: Evaluating x^sinx

    Homework Statement lim x--->0 |x|^sinx is? Homework Equations lim x-->0 f(x)^g(x), if both functions tend to 0, limit is equal to e^log[f(x).g(x)] with the same limit..(i) The Attempt at a Solution when x>0, it is x^sinx and x<0 it is -1/x^sinx. putting the first case in (i) we get...
  37. Adesh

    I How to convert the limit of a series into an integral?

    If I have a limit of a series then how can I convert it into integral. I know to convert a sum into an integral there must be Δx multiplied to each term and this must go zero. Can you please explain me the conversion of limit of series (normal series with no Δx) into an integral. Thank you.
  38. Kenneth1997

    How to solve a limit in two variables with an indeterminate form at (0,0)?

    Homework Statement lim (x,y)->(0,0) (ln(1+2x^2+y^2))/(x^2+3y^2)^2 Homework Equations The Attempt at a Solution i've been tought that i have to find another equation always bigger than this one that goes to 0 at (0,0) to find a solution. or if the solution doesn't exist, try to find two paths...
  39. E

    Can a Web Developer Fix Out-of-Profession Software Issues?

    Can a programmer "for example a web developer" understand the code and algorithm of a program which is out of his profession for example an office program stopped working or a menu/function of that office program does not work, can a web developer estimate the problem and fix it or only he can...
  40. binbagsss

    Limit , circular orbit ,schwarzschild s-t ,

    Homework Statement [/B] To take the ##lim J \to \infty ##, what are the two roots of ##r_c## in this case... So I believe it says' ##J## big enough it had 2 solutions' is basically saying just avoiding the imaginary solutions i.e. ## J^4 \geq 12G^2M^2J^2 ## (equality for one route obvs)...
  41. NihalRi

    What is the proof for the limit superior?

    Homework Statement 2. Relevant equation Below is the definition of the limit superior The Attempt at a Solution I tried to start by considering two cases, case 1 in which the sequence does not converge and case 2 in which the sequence converges and got stuck with the second case. I know...
  42. Y

    Help Taking the Limit as K goes to infinity

    Homework Statement Evaluate the limit as K goes to infinity of s_1,2 (K) Homework EquationsThe Attempt at a Solution Apparently my value for plus the square root is incorrect, apparently the correct answer is 1. Apparently my value for minus the square root is correct, it's negative...
  43. shrub_broom

    Prove that the limit of a sequence exists

    Homework Statement suppose that 0≤xm+n≤xm+xn for all m,n∈ℕ, prove that the limit of xn/n exists when n tends to infinity. Homework EquationsThe Attempt at a Solution I get that xn is bounded by zero and x1. And I guess that xn is monotonous but i find it hard to prove. Or maybe there is...
  44. D

    Approximations with the Finite Square Well

    Homework Statement Consider the standard square well potential $$V(x) = \begin{cases} -V_0 & |x| \leq a \\ 0 & |x| > a \end{cases} $$ With ##V_0 > 0##, and the wavefunctions for an even state $$\psi(x) = \begin{cases} \frac{1}{\sqrt{a}}cos(kx) & |x| \leq a \\...
  45. Mr Davis 97

    Showing that a limit does not exist

    Homework Statement Let ##f(x) = 0## if ##x## is rational and ##=1## of ##x## is irrational. Prove that ##\lim_{x\to a} f(x)## does not exist for any ##a##. Homework EquationsThe Attempt at a Solution I need help setting this one up. I was thinking that maybe I can argue by contradiction and...
  46. J

    MHB Limit of Function at x=0: Does Not Exist (DNE)

    hello I have an exercise which says: Evaluate the following limit. Enter -I if your answer is −∞, enter I if your answer is ∞, and enter DNE if the limit does not exist. limx→0[(1/(7x)−(1)/((e^(7x))−1)] e power 7x when I follow the graph for 1/7x the limit does not exist (goes to...
  47. M

    I Proving the limit of a sequence from the definition of limit

    Say that we are asked to prove, using the definition of limits, that the sequence ##\frac{4n^2+3}{n^2+n+2}## tends to ##4## as ##n## tends to infinity. The following is a screenshot of the solution I found in a YouTube video: (Note that in the definition above, "g" denotes the limit - in this...
  48. Mr Davis 97

    I Result regarding signs and a limit

    Suppose that ##f : \mathbb{R} \to \mathbb{R}## is differentiable at ##a\in\mathbb{R}##. Is it true that if ##\lim_{x\to a}\frac{f(x)-f(a)}{x-a}>0## and ##x>a## then ##f(x)>f(a)##? I'm trying to find a counterexample to show that its false because I think it is, but I'm having a hard tome doing...
  49. K

    Limit of Taylor Polynomial for Tn(x) as n Approaches Infinity

    Homework Statement Let Tn(x)=1+2x+3x^2+...+nx^(n-1) Find the value of the limit lim n->infinity Tn(1/8).The Attempt at a Solution How do I solve this? I know how to write the polynomial as a series, but not sure how if this is the best way of finding the limit.
Back
Top